A non-noetherian ring with noetherian spectrum











up vote
18
down vote

favorite
15












Question 1: Does such a ring can be found?



Note: The definition of a noetherian topological space is similar to that in rings or sets. Every descending chain of closed subsets stops after a finite number of steps



(Question 2: is this equivalent to saying that every descending chain of opens stops?)










share|cite|improve this question
























  • Question 2 arises because of Z, noetherian but non-artinian: definitions given dually deceive me every time...
    – Fosco Loregian
    Oct 21 '10 at 9:00






  • 2




    If $m$ is a maximal ideal of a commutative ring $R$, then $R / m^r$ has only $m/m^r$ as prime ideal, since any prime has the form $p/m^r$, so that $m = sqrt{m^r} subset p implies m=p$, since $m$ is maximal. So it is sufficient to find an example where $m/m^r$ is not finitely generated as $R/m^r$-module, which implies that $R/m^r$ is not Noetherian. $tag*{}$ Moreover, the spectrum of a commutative ring is Noetherian iff the ring satisfies a.c.c. on radical ideals.
    – Watson
    Jan 25 '17 at 20:55















up vote
18
down vote

favorite
15












Question 1: Does such a ring can be found?



Note: The definition of a noetherian topological space is similar to that in rings or sets. Every descending chain of closed subsets stops after a finite number of steps



(Question 2: is this equivalent to saying that every descending chain of opens stops?)










share|cite|improve this question
























  • Question 2 arises because of Z, noetherian but non-artinian: definitions given dually deceive me every time...
    – Fosco Loregian
    Oct 21 '10 at 9:00






  • 2




    If $m$ is a maximal ideal of a commutative ring $R$, then $R / m^r$ has only $m/m^r$ as prime ideal, since any prime has the form $p/m^r$, so that $m = sqrt{m^r} subset p implies m=p$, since $m$ is maximal. So it is sufficient to find an example where $m/m^r$ is not finitely generated as $R/m^r$-module, which implies that $R/m^r$ is not Noetherian. $tag*{}$ Moreover, the spectrum of a commutative ring is Noetherian iff the ring satisfies a.c.c. on radical ideals.
    – Watson
    Jan 25 '17 at 20:55













up vote
18
down vote

favorite
15









up vote
18
down vote

favorite
15






15





Question 1: Does such a ring can be found?



Note: The definition of a noetherian topological space is similar to that in rings or sets. Every descending chain of closed subsets stops after a finite number of steps



(Question 2: is this equivalent to saying that every descending chain of opens stops?)










share|cite|improve this question















Question 1: Does such a ring can be found?



Note: The definition of a noetherian topological space is similar to that in rings or sets. Every descending chain of closed subsets stops after a finite number of steps



(Question 2: is this equivalent to saying that every descending chain of opens stops?)







commutative-algebra






share|cite|improve this question















share|cite|improve this question













share|cite|improve this question




share|cite|improve this question








edited Dec 3 '17 at 8:46









user26857

39.1k123882




39.1k123882










asked Oct 21 '10 at 8:56









Fosco Loregian

4,64011946




4,64011946












  • Question 2 arises because of Z, noetherian but non-artinian: definitions given dually deceive me every time...
    – Fosco Loregian
    Oct 21 '10 at 9:00






  • 2




    If $m$ is a maximal ideal of a commutative ring $R$, then $R / m^r$ has only $m/m^r$ as prime ideal, since any prime has the form $p/m^r$, so that $m = sqrt{m^r} subset p implies m=p$, since $m$ is maximal. So it is sufficient to find an example where $m/m^r$ is not finitely generated as $R/m^r$-module, which implies that $R/m^r$ is not Noetherian. $tag*{}$ Moreover, the spectrum of a commutative ring is Noetherian iff the ring satisfies a.c.c. on radical ideals.
    – Watson
    Jan 25 '17 at 20:55


















  • Question 2 arises because of Z, noetherian but non-artinian: definitions given dually deceive me every time...
    – Fosco Loregian
    Oct 21 '10 at 9:00






  • 2




    If $m$ is a maximal ideal of a commutative ring $R$, then $R / m^r$ has only $m/m^r$ as prime ideal, since any prime has the form $p/m^r$, so that $m = sqrt{m^r} subset p implies m=p$, since $m$ is maximal. So it is sufficient to find an example where $m/m^r$ is not finitely generated as $R/m^r$-module, which implies that $R/m^r$ is not Noetherian. $tag*{}$ Moreover, the spectrum of a commutative ring is Noetherian iff the ring satisfies a.c.c. on radical ideals.
    – Watson
    Jan 25 '17 at 20:55
















Question 2 arises because of Z, noetherian but non-artinian: definitions given dually deceive me every time...
– Fosco Loregian
Oct 21 '10 at 9:00




Question 2 arises because of Z, noetherian but non-artinian: definitions given dually deceive me every time...
– Fosco Loregian
Oct 21 '10 at 9:00




2




2




If $m$ is a maximal ideal of a commutative ring $R$, then $R / m^r$ has only $m/m^r$ as prime ideal, since any prime has the form $p/m^r$, so that $m = sqrt{m^r} subset p implies m=p$, since $m$ is maximal. So it is sufficient to find an example where $m/m^r$ is not finitely generated as $R/m^r$-module, which implies that $R/m^r$ is not Noetherian. $tag*{}$ Moreover, the spectrum of a commutative ring is Noetherian iff the ring satisfies a.c.c. on radical ideals.
– Watson
Jan 25 '17 at 20:55




If $m$ is a maximal ideal of a commutative ring $R$, then $R / m^r$ has only $m/m^r$ as prime ideal, since any prime has the form $p/m^r$, so that $m = sqrt{m^r} subset p implies m=p$, since $m$ is maximal. So it is sufficient to find an example where $m/m^r$ is not finitely generated as $R/m^r$-module, which implies that $R/m^r$ is not Noetherian. $tag*{}$ Moreover, the spectrum of a commutative ring is Noetherian iff the ring satisfies a.c.c. on radical ideals.
– Watson
Jan 25 '17 at 20:55










6 Answers
6






active

oldest

votes

















up vote
33
down vote



accepted










It is easy to come up with examples when you keep in mind that $X$ and $X_{text{red}}$ are homeomorphic: The topology is not changed when you divide out (all) nilpotent sections.



For example, $A = k[x_1,x_2,...] / (x_1^2,x_2^2,...)$ satisfies $A/mathrm{rad}(A)=k$, so that $mathrm{Spec}(A)$ consists of exactly one point, namely the maximal ideal $(x_1,x_2,...)$, which is not finitely generated.



In general, $mathrm{Spec}(A)$ is a noetherian topological space iff $A$ satisfies the ascending chain condition for radical ideals.






share|cite|improve this answer























  • I was going to leave an answer to this question, but this is exactly what I was going to write!
    – Pete L. Clark
    Oct 21 '10 at 14:42


















up vote
20
down vote













If the non-noetherianness of the ring is hidden inside the nilradical, then $mathrm{Spec}$ won't see it.



Let $k$ be any ring, and let $V$ be a free $k$-module of infinite rank. Consider $R=koplus V$, and turn it into a ring by defining $$(a,v)cdot(b,w)=(ab,aw+bv).$$ (Representation-people call this a trivial extension) Then $R$ is not-noetherian, because every $k$-submodule of $V$ is an ideal in $R$. Yet $V$ is contained in the nilradical of $R$: if you look at $mathrm{Spec};R$ and at $mathrm{Spec};k$, you'll see that they are very similar.



As for your second question: no. If $k$ is an infinite field, then $mathrm{Spec};k[X]$ is noetherian, yet you'll easily find a decreasing chain of open sets init which does not stop. (What examples did you consider before asking the question? :) )






share|cite|improve this answer























  • I hope you agree that this dot looks better.
    – Rasmus
    Oct 21 '10 at 16:33


















up vote
9
down vote













Answer to question 1: Yes.



Take a non-noetherian valuation domain $R$ of finite Krull dimension. The spectrum of $R$ is totally ordered under inclusion hence finite.



H






share|cite|improve this answer




























    up vote
    3
    down vote













    Two more examples I think work...




    1. Let $k$ be a field, and $A = k[x_1,x_2,ldots]$
      a polynomial ring over $k$ in countably many indeterminates.
      Let $mathfrak{b}$ be the ideal generated by $x_1^2$ and
      $x_n - x_{n+1}^2$ for all $n geq 1$.
      Write $y_n = bar x_n$ in $B = A/mathfrak{b}$.
      Then $y_1^2 = 0$ and $y_n = y_{n+1}^2$ for all $n geq 1$,
      so $y_n^{2^n} = 0$.
      If $mathfrak{p} = (y_1,y_2,ldots)$,
      we have $B/mathfrak{p} cong k$, so $mathfrak{p}$ is maximal.
      On the other hand, the generators of $mathfrak{p}$ are nilpotent, so
      $mathfrak{p}$ is contained in the nilradical of $B$, and hence is the unique minimal prime as well.
      Since all primes then contain the maximal ideal $mathfrak{p}$
      $mathfrak{p}$ is the only prime of $B$.
      Thus $mathrm{Spec}(B) = {mathfrak{p}}$ is obviously Noetherian.
      But $(y_1) subsetneq (y_2) subsetneq (y_3) subsetneq cdots$
      is an infinite ascending chain of ideals, so $B$ is not Noetherian.


    2. Let $k$ be a field, $mathbb{Q}$ the additive group of rational numbers, and $k[mathbb{Q}]$ the group algebra. If $K$ is its field of fractions, there is a naturally associated valuation $vcolon K^times twoheadrightarrow mathbb{Q}$; let $A = {0} cup {x in K : v(x) geq 0}$ be the associated valuation ring. It has an ideal $mathfrak{a}_q = {x in K : v(x) geq q}$ for each rational $q > 0$, with $mathfrak{a}_q subsetneq mathfrak{a}_r$ if $r < q$. Thus, taking an infinite decreasing sequence $(q_j)$, we see $A$ is not Noetherian. However, the only prime ideal of $A$ is $mathfrak{p} = {x in K : v(x) > 0}$.







    share|cite|improve this answer




























      up vote
      3
      down vote













      $newcommand{Z}{mathbb{Z}}$
      Just to supply another example. Let $R$ be the subring $Z/4+(2)subset(Z/4)[x_1,x_2,cdots]$. (In general, the sum of a subring and an ideal is a subring.) $R$ consists of all polynomials in $(Z/4)[x_1,x_2,cdots]$ with coefficients of non-constant terms nilpotent. The nilradical of $R$ is $(2)$, and $R/(2)=Z/2$ is a field, so $mathrm{Spec}(R)$ is Noetherian. However, $(2x_1)subset(2x_1,2x_2)subsetcdots$ is a strictly ascending chain.






      share|cite|improve this answer






























        up vote
        0
        down vote













        Yet another example from the $p$-adic world: Let $p$ be prime $F = ℚ_p^{mathrm{tr}}$ the maximal totally ramified extension of $ℚ_p$. Then $F$ is a non-archimedically valued field with a value $lvert, ·,rvert colon F → [0..∞)$ that extends every value of every finite totally ramified extension of $ℚ_p$.



        Hence, $mathfrak o = {x ∈ F;~lvert x rvert ≤ 1 }$ is a local ring with its maximal ideal given by $mathfrak m = {x ∈ mathfrak o;~lvert x rvert < 1 }$. Let $mathfrak a = pmathfrak o = {x ∈ mathfrak o;~lvert x rvert ≤ 1/p }$. Then $operatorname{rad} mathfrak a = mathfrak m$, and so $operatorname{Spec} mathfrak o / mathfrak a = operatorname{Spec} mathfrak o / mathfrak m$ is a point. However, $mathfrak o / mathfrak a$ is not Noetherian, since there are infinite strictly ascending chains of ideals above $mathfrak a$, for instance
        $$mathfrak a = (p) subsetneq (sqrt p) subsetneq (sqrt[4] p) subsetneq (sqrt[8] p) subsetneq ….$$






        share|cite|improve this answer





















          Your Answer





          StackExchange.ifUsing("editor", function () {
          return StackExchange.using("mathjaxEditing", function () {
          StackExchange.MarkdownEditor.creationCallbacks.add(function (editor, postfix) {
          StackExchange.mathjaxEditing.prepareWmdForMathJax(editor, postfix, [["$", "$"], ["\\(","\\)"]]);
          });
          });
          }, "mathjax-editing");

          StackExchange.ready(function() {
          var channelOptions = {
          tags: "".split(" "),
          id: "69"
          };
          initTagRenderer("".split(" "), "".split(" "), channelOptions);

          StackExchange.using("externalEditor", function() {
          // Have to fire editor after snippets, if snippets enabled
          if (StackExchange.settings.snippets.snippetsEnabled) {
          StackExchange.using("snippets", function() {
          createEditor();
          });
          }
          else {
          createEditor();
          }
          });

          function createEditor() {
          StackExchange.prepareEditor({
          heartbeatType: 'answer',
          convertImagesToLinks: true,
          noModals: true,
          showLowRepImageUploadWarning: true,
          reputationToPostImages: 10,
          bindNavPrevention: true,
          postfix: "",
          imageUploader: {
          brandingHtml: "Powered by u003ca class="icon-imgur-white" href="https://imgur.com/"u003eu003c/au003e",
          contentPolicyHtml: "User contributions licensed under u003ca href="https://creativecommons.org/licenses/by-sa/3.0/"u003ecc by-sa 3.0 with attribution requiredu003c/au003e u003ca href="https://stackoverflow.com/legal/content-policy"u003e(content policy)u003c/au003e",
          allowUrls: true
          },
          noCode: true, onDemand: true,
          discardSelector: ".discard-answer"
          ,immediatelyShowMarkdownHelp:true
          });


          }
          });














          draft saved

          draft discarded


















          StackExchange.ready(
          function () {
          StackExchange.openid.initPostLogin('.new-post-login', 'https%3a%2f%2fmath.stackexchange.com%2fquestions%2f7392%2fa-non-noetherian-ring-with-noetherian-spectrum%23new-answer', 'question_page');
          }
          );

          Post as a guest















          Required, but never shown

























          6 Answers
          6






          active

          oldest

          votes








          6 Answers
          6






          active

          oldest

          votes









          active

          oldest

          votes






          active

          oldest

          votes








          up vote
          33
          down vote



          accepted










          It is easy to come up with examples when you keep in mind that $X$ and $X_{text{red}}$ are homeomorphic: The topology is not changed when you divide out (all) nilpotent sections.



          For example, $A = k[x_1,x_2,...] / (x_1^2,x_2^2,...)$ satisfies $A/mathrm{rad}(A)=k$, so that $mathrm{Spec}(A)$ consists of exactly one point, namely the maximal ideal $(x_1,x_2,...)$, which is not finitely generated.



          In general, $mathrm{Spec}(A)$ is a noetherian topological space iff $A$ satisfies the ascending chain condition for radical ideals.






          share|cite|improve this answer























          • I was going to leave an answer to this question, but this is exactly what I was going to write!
            – Pete L. Clark
            Oct 21 '10 at 14:42















          up vote
          33
          down vote



          accepted










          It is easy to come up with examples when you keep in mind that $X$ and $X_{text{red}}$ are homeomorphic: The topology is not changed when you divide out (all) nilpotent sections.



          For example, $A = k[x_1,x_2,...] / (x_1^2,x_2^2,...)$ satisfies $A/mathrm{rad}(A)=k$, so that $mathrm{Spec}(A)$ consists of exactly one point, namely the maximal ideal $(x_1,x_2,...)$, which is not finitely generated.



          In general, $mathrm{Spec}(A)$ is a noetherian topological space iff $A$ satisfies the ascending chain condition for radical ideals.






          share|cite|improve this answer























          • I was going to leave an answer to this question, but this is exactly what I was going to write!
            – Pete L. Clark
            Oct 21 '10 at 14:42













          up vote
          33
          down vote



          accepted







          up vote
          33
          down vote



          accepted






          It is easy to come up with examples when you keep in mind that $X$ and $X_{text{red}}$ are homeomorphic: The topology is not changed when you divide out (all) nilpotent sections.



          For example, $A = k[x_1,x_2,...] / (x_1^2,x_2^2,...)$ satisfies $A/mathrm{rad}(A)=k$, so that $mathrm{Spec}(A)$ consists of exactly one point, namely the maximal ideal $(x_1,x_2,...)$, which is not finitely generated.



          In general, $mathrm{Spec}(A)$ is a noetherian topological space iff $A$ satisfies the ascending chain condition for radical ideals.






          share|cite|improve this answer














          It is easy to come up with examples when you keep in mind that $X$ and $X_{text{red}}$ are homeomorphic: The topology is not changed when you divide out (all) nilpotent sections.



          For example, $A = k[x_1,x_2,...] / (x_1^2,x_2^2,...)$ satisfies $A/mathrm{rad}(A)=k$, so that $mathrm{Spec}(A)$ consists of exactly one point, namely the maximal ideal $(x_1,x_2,...)$, which is not finitely generated.



          In general, $mathrm{Spec}(A)$ is a noetherian topological space iff $A$ satisfies the ascending chain condition for radical ideals.







          share|cite|improve this answer














          share|cite|improve this answer



          share|cite|improve this answer








          edited Dec 3 '17 at 8:48









          user26857

          39.1k123882




          39.1k123882










          answered Oct 21 '10 at 14:30









          Martin Brandenburg

          107k13155324




          107k13155324












          • I was going to leave an answer to this question, but this is exactly what I was going to write!
            – Pete L. Clark
            Oct 21 '10 at 14:42


















          • I was going to leave an answer to this question, but this is exactly what I was going to write!
            – Pete L. Clark
            Oct 21 '10 at 14:42
















          I was going to leave an answer to this question, but this is exactly what I was going to write!
          – Pete L. Clark
          Oct 21 '10 at 14:42




          I was going to leave an answer to this question, but this is exactly what I was going to write!
          – Pete L. Clark
          Oct 21 '10 at 14:42










          up vote
          20
          down vote













          If the non-noetherianness of the ring is hidden inside the nilradical, then $mathrm{Spec}$ won't see it.



          Let $k$ be any ring, and let $V$ be a free $k$-module of infinite rank. Consider $R=koplus V$, and turn it into a ring by defining $$(a,v)cdot(b,w)=(ab,aw+bv).$$ (Representation-people call this a trivial extension) Then $R$ is not-noetherian, because every $k$-submodule of $V$ is an ideal in $R$. Yet $V$ is contained in the nilradical of $R$: if you look at $mathrm{Spec};R$ and at $mathrm{Spec};k$, you'll see that they are very similar.



          As for your second question: no. If $k$ is an infinite field, then $mathrm{Spec};k[X]$ is noetherian, yet you'll easily find a decreasing chain of open sets init which does not stop. (What examples did you consider before asking the question? :) )






          share|cite|improve this answer























          • I hope you agree that this dot looks better.
            – Rasmus
            Oct 21 '10 at 16:33















          up vote
          20
          down vote













          If the non-noetherianness of the ring is hidden inside the nilradical, then $mathrm{Spec}$ won't see it.



          Let $k$ be any ring, and let $V$ be a free $k$-module of infinite rank. Consider $R=koplus V$, and turn it into a ring by defining $$(a,v)cdot(b,w)=(ab,aw+bv).$$ (Representation-people call this a trivial extension) Then $R$ is not-noetherian, because every $k$-submodule of $V$ is an ideal in $R$. Yet $V$ is contained in the nilradical of $R$: if you look at $mathrm{Spec};R$ and at $mathrm{Spec};k$, you'll see that they are very similar.



          As for your second question: no. If $k$ is an infinite field, then $mathrm{Spec};k[X]$ is noetherian, yet you'll easily find a decreasing chain of open sets init which does not stop. (What examples did you consider before asking the question? :) )






          share|cite|improve this answer























          • I hope you agree that this dot looks better.
            – Rasmus
            Oct 21 '10 at 16:33













          up vote
          20
          down vote










          up vote
          20
          down vote









          If the non-noetherianness of the ring is hidden inside the nilradical, then $mathrm{Spec}$ won't see it.



          Let $k$ be any ring, and let $V$ be a free $k$-module of infinite rank. Consider $R=koplus V$, and turn it into a ring by defining $$(a,v)cdot(b,w)=(ab,aw+bv).$$ (Representation-people call this a trivial extension) Then $R$ is not-noetherian, because every $k$-submodule of $V$ is an ideal in $R$. Yet $V$ is contained in the nilradical of $R$: if you look at $mathrm{Spec};R$ and at $mathrm{Spec};k$, you'll see that they are very similar.



          As for your second question: no. If $k$ is an infinite field, then $mathrm{Spec};k[X]$ is noetherian, yet you'll easily find a decreasing chain of open sets init which does not stop. (What examples did you consider before asking the question? :) )






          share|cite|improve this answer














          If the non-noetherianness of the ring is hidden inside the nilradical, then $mathrm{Spec}$ won't see it.



          Let $k$ be any ring, and let $V$ be a free $k$-module of infinite rank. Consider $R=koplus V$, and turn it into a ring by defining $$(a,v)cdot(b,w)=(ab,aw+bv).$$ (Representation-people call this a trivial extension) Then $R$ is not-noetherian, because every $k$-submodule of $V$ is an ideal in $R$. Yet $V$ is contained in the nilradical of $R$: if you look at $mathrm{Spec};R$ and at $mathrm{Spec};k$, you'll see that they are very similar.



          As for your second question: no. If $k$ is an infinite field, then $mathrm{Spec};k[X]$ is noetherian, yet you'll easily find a decreasing chain of open sets init which does not stop. (What examples did you consider before asking the question? :) )







          share|cite|improve this answer














          share|cite|improve this answer



          share|cite|improve this answer








          edited Oct 21 '10 at 17:00

























          answered Oct 21 '10 at 14:03









          Mariano Suárez-Álvarez

          110k7155280




          110k7155280












          • I hope you agree that this dot looks better.
            – Rasmus
            Oct 21 '10 at 16:33


















          • I hope you agree that this dot looks better.
            – Rasmus
            Oct 21 '10 at 16:33
















          I hope you agree that this dot looks better.
          – Rasmus
          Oct 21 '10 at 16:33




          I hope you agree that this dot looks better.
          – Rasmus
          Oct 21 '10 at 16:33










          up vote
          9
          down vote













          Answer to question 1: Yes.



          Take a non-noetherian valuation domain $R$ of finite Krull dimension. The spectrum of $R$ is totally ordered under inclusion hence finite.



          H






          share|cite|improve this answer

























            up vote
            9
            down vote













            Answer to question 1: Yes.



            Take a non-noetherian valuation domain $R$ of finite Krull dimension. The spectrum of $R$ is totally ordered under inclusion hence finite.



            H






            share|cite|improve this answer























              up vote
              9
              down vote










              up vote
              9
              down vote









              Answer to question 1: Yes.



              Take a non-noetherian valuation domain $R$ of finite Krull dimension. The spectrum of $R$ is totally ordered under inclusion hence finite.



              H






              share|cite|improve this answer












              Answer to question 1: Yes.



              Take a non-noetherian valuation domain $R$ of finite Krull dimension. The spectrum of $R$ is totally ordered under inclusion hence finite.



              H







              share|cite|improve this answer












              share|cite|improve this answer



              share|cite|improve this answer










              answered Oct 21 '10 at 12:27









              Hagen Knaf

              6,8521317




              6,8521317






















                  up vote
                  3
                  down vote













                  Two more examples I think work...




                  1. Let $k$ be a field, and $A = k[x_1,x_2,ldots]$
                    a polynomial ring over $k$ in countably many indeterminates.
                    Let $mathfrak{b}$ be the ideal generated by $x_1^2$ and
                    $x_n - x_{n+1}^2$ for all $n geq 1$.
                    Write $y_n = bar x_n$ in $B = A/mathfrak{b}$.
                    Then $y_1^2 = 0$ and $y_n = y_{n+1}^2$ for all $n geq 1$,
                    so $y_n^{2^n} = 0$.
                    If $mathfrak{p} = (y_1,y_2,ldots)$,
                    we have $B/mathfrak{p} cong k$, so $mathfrak{p}$ is maximal.
                    On the other hand, the generators of $mathfrak{p}$ are nilpotent, so
                    $mathfrak{p}$ is contained in the nilradical of $B$, and hence is the unique minimal prime as well.
                    Since all primes then contain the maximal ideal $mathfrak{p}$
                    $mathfrak{p}$ is the only prime of $B$.
                    Thus $mathrm{Spec}(B) = {mathfrak{p}}$ is obviously Noetherian.
                    But $(y_1) subsetneq (y_2) subsetneq (y_3) subsetneq cdots$
                    is an infinite ascending chain of ideals, so $B$ is not Noetherian.


                  2. Let $k$ be a field, $mathbb{Q}$ the additive group of rational numbers, and $k[mathbb{Q}]$ the group algebra. If $K$ is its field of fractions, there is a naturally associated valuation $vcolon K^times twoheadrightarrow mathbb{Q}$; let $A = {0} cup {x in K : v(x) geq 0}$ be the associated valuation ring. It has an ideal $mathfrak{a}_q = {x in K : v(x) geq q}$ for each rational $q > 0$, with $mathfrak{a}_q subsetneq mathfrak{a}_r$ if $r < q$. Thus, taking an infinite decreasing sequence $(q_j)$, we see $A$ is not Noetherian. However, the only prime ideal of $A$ is $mathfrak{p} = {x in K : v(x) > 0}$.







                  share|cite|improve this answer

























                    up vote
                    3
                    down vote













                    Two more examples I think work...




                    1. Let $k$ be a field, and $A = k[x_1,x_2,ldots]$
                      a polynomial ring over $k$ in countably many indeterminates.
                      Let $mathfrak{b}$ be the ideal generated by $x_1^2$ and
                      $x_n - x_{n+1}^2$ for all $n geq 1$.
                      Write $y_n = bar x_n$ in $B = A/mathfrak{b}$.
                      Then $y_1^2 = 0$ and $y_n = y_{n+1}^2$ for all $n geq 1$,
                      so $y_n^{2^n} = 0$.
                      If $mathfrak{p} = (y_1,y_2,ldots)$,
                      we have $B/mathfrak{p} cong k$, so $mathfrak{p}$ is maximal.
                      On the other hand, the generators of $mathfrak{p}$ are nilpotent, so
                      $mathfrak{p}$ is contained in the nilradical of $B$, and hence is the unique minimal prime as well.
                      Since all primes then contain the maximal ideal $mathfrak{p}$
                      $mathfrak{p}$ is the only prime of $B$.
                      Thus $mathrm{Spec}(B) = {mathfrak{p}}$ is obviously Noetherian.
                      But $(y_1) subsetneq (y_2) subsetneq (y_3) subsetneq cdots$
                      is an infinite ascending chain of ideals, so $B$ is not Noetherian.


                    2. Let $k$ be a field, $mathbb{Q}$ the additive group of rational numbers, and $k[mathbb{Q}]$ the group algebra. If $K$ is its field of fractions, there is a naturally associated valuation $vcolon K^times twoheadrightarrow mathbb{Q}$; let $A = {0} cup {x in K : v(x) geq 0}$ be the associated valuation ring. It has an ideal $mathfrak{a}_q = {x in K : v(x) geq q}$ for each rational $q > 0$, with $mathfrak{a}_q subsetneq mathfrak{a}_r$ if $r < q$. Thus, taking an infinite decreasing sequence $(q_j)$, we see $A$ is not Noetherian. However, the only prime ideal of $A$ is $mathfrak{p} = {x in K : v(x) > 0}$.







                    share|cite|improve this answer























                      up vote
                      3
                      down vote










                      up vote
                      3
                      down vote









                      Two more examples I think work...




                      1. Let $k$ be a field, and $A = k[x_1,x_2,ldots]$
                        a polynomial ring over $k$ in countably many indeterminates.
                        Let $mathfrak{b}$ be the ideal generated by $x_1^2$ and
                        $x_n - x_{n+1}^2$ for all $n geq 1$.
                        Write $y_n = bar x_n$ in $B = A/mathfrak{b}$.
                        Then $y_1^2 = 0$ and $y_n = y_{n+1}^2$ for all $n geq 1$,
                        so $y_n^{2^n} = 0$.
                        If $mathfrak{p} = (y_1,y_2,ldots)$,
                        we have $B/mathfrak{p} cong k$, so $mathfrak{p}$ is maximal.
                        On the other hand, the generators of $mathfrak{p}$ are nilpotent, so
                        $mathfrak{p}$ is contained in the nilradical of $B$, and hence is the unique minimal prime as well.
                        Since all primes then contain the maximal ideal $mathfrak{p}$
                        $mathfrak{p}$ is the only prime of $B$.
                        Thus $mathrm{Spec}(B) = {mathfrak{p}}$ is obviously Noetherian.
                        But $(y_1) subsetneq (y_2) subsetneq (y_3) subsetneq cdots$
                        is an infinite ascending chain of ideals, so $B$ is not Noetherian.


                      2. Let $k$ be a field, $mathbb{Q}$ the additive group of rational numbers, and $k[mathbb{Q}]$ the group algebra. If $K$ is its field of fractions, there is a naturally associated valuation $vcolon K^times twoheadrightarrow mathbb{Q}$; let $A = {0} cup {x in K : v(x) geq 0}$ be the associated valuation ring. It has an ideal $mathfrak{a}_q = {x in K : v(x) geq q}$ for each rational $q > 0$, with $mathfrak{a}_q subsetneq mathfrak{a}_r$ if $r < q$. Thus, taking an infinite decreasing sequence $(q_j)$, we see $A$ is not Noetherian. However, the only prime ideal of $A$ is $mathfrak{p} = {x in K : v(x) > 0}$.







                      share|cite|improve this answer












                      Two more examples I think work...




                      1. Let $k$ be a field, and $A = k[x_1,x_2,ldots]$
                        a polynomial ring over $k$ in countably many indeterminates.
                        Let $mathfrak{b}$ be the ideal generated by $x_1^2$ and
                        $x_n - x_{n+1}^2$ for all $n geq 1$.
                        Write $y_n = bar x_n$ in $B = A/mathfrak{b}$.
                        Then $y_1^2 = 0$ and $y_n = y_{n+1}^2$ for all $n geq 1$,
                        so $y_n^{2^n} = 0$.
                        If $mathfrak{p} = (y_1,y_2,ldots)$,
                        we have $B/mathfrak{p} cong k$, so $mathfrak{p}$ is maximal.
                        On the other hand, the generators of $mathfrak{p}$ are nilpotent, so
                        $mathfrak{p}$ is contained in the nilradical of $B$, and hence is the unique minimal prime as well.
                        Since all primes then contain the maximal ideal $mathfrak{p}$
                        $mathfrak{p}$ is the only prime of $B$.
                        Thus $mathrm{Spec}(B) = {mathfrak{p}}$ is obviously Noetherian.
                        But $(y_1) subsetneq (y_2) subsetneq (y_3) subsetneq cdots$
                        is an infinite ascending chain of ideals, so $B$ is not Noetherian.


                      2. Let $k$ be a field, $mathbb{Q}$ the additive group of rational numbers, and $k[mathbb{Q}]$ the group algebra. If $K$ is its field of fractions, there is a naturally associated valuation $vcolon K^times twoheadrightarrow mathbb{Q}$; let $A = {0} cup {x in K : v(x) geq 0}$ be the associated valuation ring. It has an ideal $mathfrak{a}_q = {x in K : v(x) geq q}$ for each rational $q > 0$, with $mathfrak{a}_q subsetneq mathfrak{a}_r$ if $r < q$. Thus, taking an infinite decreasing sequence $(q_j)$, we see $A$ is not Noetherian. However, the only prime ideal of $A$ is $mathfrak{p} = {x in K : v(x) > 0}$.








                      share|cite|improve this answer












                      share|cite|improve this answer



                      share|cite|improve this answer










                      answered Feb 15 '11 at 14:14









                      jdc

                      2,7311228




                      2,7311228






















                          up vote
                          3
                          down vote













                          $newcommand{Z}{mathbb{Z}}$
                          Just to supply another example. Let $R$ be the subring $Z/4+(2)subset(Z/4)[x_1,x_2,cdots]$. (In general, the sum of a subring and an ideal is a subring.) $R$ consists of all polynomials in $(Z/4)[x_1,x_2,cdots]$ with coefficients of non-constant terms nilpotent. The nilradical of $R$ is $(2)$, and $R/(2)=Z/2$ is a field, so $mathrm{Spec}(R)$ is Noetherian. However, $(2x_1)subset(2x_1,2x_2)subsetcdots$ is a strictly ascending chain.






                          share|cite|improve this answer



























                            up vote
                            3
                            down vote













                            $newcommand{Z}{mathbb{Z}}$
                            Just to supply another example. Let $R$ be the subring $Z/4+(2)subset(Z/4)[x_1,x_2,cdots]$. (In general, the sum of a subring and an ideal is a subring.) $R$ consists of all polynomials in $(Z/4)[x_1,x_2,cdots]$ with coefficients of non-constant terms nilpotent. The nilradical of $R$ is $(2)$, and $R/(2)=Z/2$ is a field, so $mathrm{Spec}(R)$ is Noetherian. However, $(2x_1)subset(2x_1,2x_2)subsetcdots$ is a strictly ascending chain.






                            share|cite|improve this answer

























                              up vote
                              3
                              down vote










                              up vote
                              3
                              down vote









                              $newcommand{Z}{mathbb{Z}}$
                              Just to supply another example. Let $R$ be the subring $Z/4+(2)subset(Z/4)[x_1,x_2,cdots]$. (In general, the sum of a subring and an ideal is a subring.) $R$ consists of all polynomials in $(Z/4)[x_1,x_2,cdots]$ with coefficients of non-constant terms nilpotent. The nilradical of $R$ is $(2)$, and $R/(2)=Z/2$ is a field, so $mathrm{Spec}(R)$ is Noetherian. However, $(2x_1)subset(2x_1,2x_2)subsetcdots$ is a strictly ascending chain.






                              share|cite|improve this answer














                              $newcommand{Z}{mathbb{Z}}$
                              Just to supply another example. Let $R$ be the subring $Z/4+(2)subset(Z/4)[x_1,x_2,cdots]$. (In general, the sum of a subring and an ideal is a subring.) $R$ consists of all polynomials in $(Z/4)[x_1,x_2,cdots]$ with coefficients of non-constant terms nilpotent. The nilradical of $R$ is $(2)$, and $R/(2)=Z/2$ is a field, so $mathrm{Spec}(R)$ is Noetherian. However, $(2x_1)subset(2x_1,2x_2)subsetcdots$ is a strictly ascending chain.







                              share|cite|improve this answer














                              share|cite|improve this answer



                              share|cite|improve this answer








                              edited Mar 11 '13 at 23:41

























                              answered Mar 11 '13 at 3:50









                              Junyan Xu

                              38828




                              38828






















                                  up vote
                                  0
                                  down vote













                                  Yet another example from the $p$-adic world: Let $p$ be prime $F = ℚ_p^{mathrm{tr}}$ the maximal totally ramified extension of $ℚ_p$. Then $F$ is a non-archimedically valued field with a value $lvert, ·,rvert colon F → [0..∞)$ that extends every value of every finite totally ramified extension of $ℚ_p$.



                                  Hence, $mathfrak o = {x ∈ F;~lvert x rvert ≤ 1 }$ is a local ring with its maximal ideal given by $mathfrak m = {x ∈ mathfrak o;~lvert x rvert < 1 }$. Let $mathfrak a = pmathfrak o = {x ∈ mathfrak o;~lvert x rvert ≤ 1/p }$. Then $operatorname{rad} mathfrak a = mathfrak m$, and so $operatorname{Spec} mathfrak o / mathfrak a = operatorname{Spec} mathfrak o / mathfrak m$ is a point. However, $mathfrak o / mathfrak a$ is not Noetherian, since there are infinite strictly ascending chains of ideals above $mathfrak a$, for instance
                                  $$mathfrak a = (p) subsetneq (sqrt p) subsetneq (sqrt[4] p) subsetneq (sqrt[8] p) subsetneq ….$$






                                  share|cite|improve this answer

























                                    up vote
                                    0
                                    down vote













                                    Yet another example from the $p$-adic world: Let $p$ be prime $F = ℚ_p^{mathrm{tr}}$ the maximal totally ramified extension of $ℚ_p$. Then $F$ is a non-archimedically valued field with a value $lvert, ·,rvert colon F → [0..∞)$ that extends every value of every finite totally ramified extension of $ℚ_p$.



                                    Hence, $mathfrak o = {x ∈ F;~lvert x rvert ≤ 1 }$ is a local ring with its maximal ideal given by $mathfrak m = {x ∈ mathfrak o;~lvert x rvert < 1 }$. Let $mathfrak a = pmathfrak o = {x ∈ mathfrak o;~lvert x rvert ≤ 1/p }$. Then $operatorname{rad} mathfrak a = mathfrak m$, and so $operatorname{Spec} mathfrak o / mathfrak a = operatorname{Spec} mathfrak o / mathfrak m$ is a point. However, $mathfrak o / mathfrak a$ is not Noetherian, since there are infinite strictly ascending chains of ideals above $mathfrak a$, for instance
                                    $$mathfrak a = (p) subsetneq (sqrt p) subsetneq (sqrt[4] p) subsetneq (sqrt[8] p) subsetneq ….$$






                                    share|cite|improve this answer























                                      up vote
                                      0
                                      down vote










                                      up vote
                                      0
                                      down vote









                                      Yet another example from the $p$-adic world: Let $p$ be prime $F = ℚ_p^{mathrm{tr}}$ the maximal totally ramified extension of $ℚ_p$. Then $F$ is a non-archimedically valued field with a value $lvert, ·,rvert colon F → [0..∞)$ that extends every value of every finite totally ramified extension of $ℚ_p$.



                                      Hence, $mathfrak o = {x ∈ F;~lvert x rvert ≤ 1 }$ is a local ring with its maximal ideal given by $mathfrak m = {x ∈ mathfrak o;~lvert x rvert < 1 }$. Let $mathfrak a = pmathfrak o = {x ∈ mathfrak o;~lvert x rvert ≤ 1/p }$. Then $operatorname{rad} mathfrak a = mathfrak m$, and so $operatorname{Spec} mathfrak o / mathfrak a = operatorname{Spec} mathfrak o / mathfrak m$ is a point. However, $mathfrak o / mathfrak a$ is not Noetherian, since there are infinite strictly ascending chains of ideals above $mathfrak a$, for instance
                                      $$mathfrak a = (p) subsetneq (sqrt p) subsetneq (sqrt[4] p) subsetneq (sqrt[8] p) subsetneq ….$$






                                      share|cite|improve this answer












                                      Yet another example from the $p$-adic world: Let $p$ be prime $F = ℚ_p^{mathrm{tr}}$ the maximal totally ramified extension of $ℚ_p$. Then $F$ is a non-archimedically valued field with a value $lvert, ·,rvert colon F → [0..∞)$ that extends every value of every finite totally ramified extension of $ℚ_p$.



                                      Hence, $mathfrak o = {x ∈ F;~lvert x rvert ≤ 1 }$ is a local ring with its maximal ideal given by $mathfrak m = {x ∈ mathfrak o;~lvert x rvert < 1 }$. Let $mathfrak a = pmathfrak o = {x ∈ mathfrak o;~lvert x rvert ≤ 1/p }$. Then $operatorname{rad} mathfrak a = mathfrak m$, and so $operatorname{Spec} mathfrak o / mathfrak a = operatorname{Spec} mathfrak o / mathfrak m$ is a point. However, $mathfrak o / mathfrak a$ is not Noetherian, since there are infinite strictly ascending chains of ideals above $mathfrak a$, for instance
                                      $$mathfrak a = (p) subsetneq (sqrt p) subsetneq (sqrt[4] p) subsetneq (sqrt[8] p) subsetneq ….$$







                                      share|cite|improve this answer












                                      share|cite|improve this answer



                                      share|cite|improve this answer










                                      answered Nov 16 at 19:22









                                      k.stm

                                      10.7k22249




                                      10.7k22249






























                                          draft saved

                                          draft discarded




















































                                          Thanks for contributing an answer to Mathematics Stack Exchange!


                                          • Please be sure to answer the question. Provide details and share your research!

                                          But avoid



                                          • Asking for help, clarification, or responding to other answers.

                                          • Making statements based on opinion; back them up with references or personal experience.


                                          Use MathJax to format equations. MathJax reference.


                                          To learn more, see our tips on writing great answers.





                                          Some of your past answers have not been well-received, and you're in danger of being blocked from answering.


                                          Please pay close attention to the following guidance:


                                          • Please be sure to answer the question. Provide details and share your research!

                                          But avoid



                                          • Asking for help, clarification, or responding to other answers.

                                          • Making statements based on opinion; back them up with references or personal experience.


                                          To learn more, see our tips on writing great answers.




                                          draft saved


                                          draft discarded














                                          StackExchange.ready(
                                          function () {
                                          StackExchange.openid.initPostLogin('.new-post-login', 'https%3a%2f%2fmath.stackexchange.com%2fquestions%2f7392%2fa-non-noetherian-ring-with-noetherian-spectrum%23new-answer', 'question_page');
                                          }
                                          );

                                          Post as a guest















                                          Required, but never shown





















































                                          Required, but never shown














                                          Required, but never shown












                                          Required, but never shown







                                          Required, but never shown

































                                          Required, but never shown














                                          Required, but never shown












                                          Required, but never shown







                                          Required, but never shown







                                          Popular posts from this blog

                                          Plaza Victoria

                                          In PowerPoint, is there a keyboard shortcut for bulleted / numbered list?

                                          How to put 3 figures in Latex with 2 figures side by side and 1 below these side by side images but in...